



Study with the several resources on Docsity
Earn points by helping other students or get them with a premium plan
Prepare for your exams
Study with the several resources on Docsity
Earn points to download
Earn points by helping other students or get them with a premium plan
Community
Ask the community for help and clear up your study doubts
Discover the best universities in your country according to Docsity users
Free resources
Download our free guides on studying techniques, anxiety management strategies, and thesis advice from Docsity tutors
Seminar pdf Simon Fraser university
Typology: Lecture notes
1 / 5
This page cannot be seen from the preview
Don't miss anything!
Content:
Section 2.3: Limit Laws
Section 2.5: Continuity
Questions:
bx^2 − 10 x + 10 + b x^2 − x − 2 exists? If so, find the value of b and the value of the limit. Solution. We are interested in what happens near x = 2. In the denominator, we get:
(2)^2 − (2) − 2 = 4 − 2 − 2 = 0,
so in order for the limit to exist, the numerator must also be 0 at x = 2. If this is not the case, then as the limit approaches 2 from one side, the denominator gets smaller and smaller and the limit approaches ∞, while from the other side, the limit approaches −∞. This means that b must be such that
b(2)^2 − 10(2) + 10 + b = 4b − 20 + 10 + b = 5b − 10 = 0
which gives that b = 2. The limit is therefore,
lim x→ 2
2 x^2 − 10 x + 12 x^2 − x − 2
= lim x→ 2
2(x^2 − 5 x + 6) (x − 2)(x + 1)
= lim x→ 2
(^2) (x − 2)(x − 3) (x − 2)(x + 1) = lim x→ 2
2(x − 3) x + 1
(a) lim x→− 1 f (x)
(b) lim x→ 1 f (x)
(c) lim x→ 0 f (x)
Solution.
(a) We have the following limits:
lim x→− 1 +
x^4 = 1; lim x→− 1 −
x^4 = 1; lim x→− 1 +
x^2 = 1; lim x→− 1 −
x^2 = 1
so by application of the Squeeze Theorem we get that
lim x→− 1 f (x) = 1.
(b) We have the following limits:
lim x→ 1 +^
x^4 = 1; lim x→ 1 −^
x^4 = 1; lim x→ 1 +^
x^2 = 1; lim x→ 1 −^
x^2 = 1
so by application of the Squeeze Theorem we get that
lim x→ 1 f (x) = 1.
(c) We have the following limits: lim x→ 0 x^4 = 0; lim x→ 0 x^2 = 0;
so by application of the Squeeze Theorem we get
lim x→ 0
f (x) = 0.
(a) lim x→ 10
x^2 − 100 x − 9 (b) lim x→ 10
f (x), where f (x) = x^2 for all x ̸= 10, but f (10) = 99.
(c) lim x→ 8
√ (^3) x − 2
x − 8
(d) lim x→ 8
(x − 8)(x + 2) |x − 8 | Solution.
(a) lim x→ 10
x^2 − 100 x − 9
(b) lim x→ 10 f (x) = lim x→ 10 x^2 = 100 so lim x→ 10 f (x) = 100
(c)
lim x→ 8
√ (^3) x − 2
x − 8 = lim x→ 8
x − 2) ( √^3 x − 2)( √^3 x^2 + 2 √^3 x + 4) = lim^ x→^8
√ (^3) x (^2) + 2 √ (^3) x + 4 =
(d) For finding lim x→ 8
(x − 8)(x + 2) |x − 8 |
, we use
|x − 8 | =
x − 8 if x ≥ 8 −(x − 8) if x < 8.
Let’s consider left and right hand limits.
lim x→ 8 +^
(x − 8)(x + 2) (x^ −^ 8)^
= lim x→ 8 +
(x + 2) = 10.
lim x→ 8 −^
(x − 8)(x + 2) −(x − 8)
= lim x→ 8 −^
−(x + 2) = − 10.
Since the right and left hand limits are not equal, this limit does not exist.
lim x→ 0
p x^3 + x^2 sin
π x
Solution. Let f (x) = −
x^3 + x^2 , g(x) =
x^3 + x^2 sin(π/x), and h(x) =
x^3 + x^2. Notice the domain of f (x) and h(x) is {x ∈ R|x ≥ − 1 }, i.e. those values such that x^3 + x^2 ≥ 0. Then we know that − 1 ≤ sin(π/x) ≤ 1 and so the following holds for x ≥ − 1 :
−
p x^3 + x^2 ≤
p x^3 + x^2 sin (π/x) ≤
p x^3 + x^2 ⇒ f (x) ≤ g(x) ≤ h(x)
And since lim x→ 0 f (x) = lim x→ 0 h(x) = 0, by the Squeeze Theorem, lim x→ 0 g(x) = 0.
which means that f is continuous from the left at x = 1 for any value of b. But it also has to be continuous from the right. We find the right hand limit:
lim x→ 1 +
f (x) = lim x→ 1 +
x^2 + 2b = 1 + 2b
In order for this function to be continuous from the right, we need limx→ 1 +^ f (x) = f (1), which means that b − 1 = 1 + 2b ⇒ b = − 2. Thus, when b = − 2 , f is continuous at x = 1. □
f (x) =
(^) x (^2) − 9 x− 3 if^ x^ ̸= 3 0 if x = 3.
Observe that, for x ̸= 3,
f (x) =
x^2 − 9 x − 3
(x + 3)(x − 3) x − 3
(x + 3)(x − 3) (x − 3) =^ x^ + 3.
and conclude:
(a) The function f is continuous for on the set(−∞, 3) ∪ (3, ∞). (b) From lim x→ 3 f (x) = lim x→ 3 (x + 3) = 6 ̸= 0 = f (3)
it follows that the function f is not continuous at x = 3.
The graph of this function:
x
y
Solution. Observe that the given function if continuous on R. Also, for all x ∈ R,
− 1 ≤ sin x ≤ 1 ⇒ x^2 − 10 ≤ f (x) ≤ x^2 + 10
Since
10 ≈ 10 · 3 .15 = 31. 5 , we consider an interval that contains the number 31. 5. To keep our calculations simple, take the interval [0, 40]. Now
f (0) = 0^2 + 10 sin(0) = 0 < 1000 and f (40) = 40^2 + 10 sin(40) ≤ 1600 − 10 = 1590 > 1000.
Therefore, since f is continuous on the closed interval [0, 40] and since 1000 ∈ [f (0), f (40)] = [0, 1590], by the Intermediate Value Theorem there is a number c in (0, 40) such that f (c) = 1000.
(b) Show that the equation 2 x^ = (^10) x has no solution for x < 0. Solution. Let f (x) = 2x^ − (^10) x. Note that the domain of f is the set R{ 0 } and that on its domain, as a sum of two continuous functions, f is continuous.
(a) Since f is continuous on (0, ∞) and since lim x→ 0 +
f (x) = −∞ and lim x→∞ f (x) = ∞, but the Intermediate Value Theorem there is a ∈ (0, ∞) such that f (a) = 0. (b) For all x ∈ (−∞, 0) we have that (^10) x < 0 which implies that for all x ∈ (−∞, 0) we have that all f (x) > 0.